Series that evaluates to different valuesWhat is the sum of the infinite telescopic series $sum_n=1^infty arctan (n+4)-arctan(n+2))$?Convergence of the series using power seriesSimplify this summation: $sumlimits_k=5^inftybinomk-1k-5frack^32^k$Convergent series with general term $a_nb_n$Decide whether the series $sum_n=1^infty frac1+5^n1+6^n$ converges or divergesCan different choices of regulator assign different values to the same divergent series?Riemann series theorem - when can I be sure that my series diverges?Find for which values of $x$ the series convergesWorking on series $A(x):=sum_n=0^infty2^-ncos(nx)$Expected value of an r.v. that counts the number of pickable values that never appear in a random sequence

Why didn't the Avengers use this object earlier?

"The van's really booking"

Did galley captains put corks in the mouths of slave rowers to keep them quiet?

How to redirect stdout to a file, and stdout+stderr to another one?

How do I adjust encounters to challenge my lycanthrope players without negating their cool new abilities?

Do not cross the line!

Adding labels and comments to a matrix

Is it wrong to omit object pronouns in these sentences?

Should I communicate in my applications that I'm unemployed out of choice rather than because nobody will have me?

Why was my Canon Speedlite 600EX triggering other flashes?

What is this old US Air Force plane?

How to make a not so good looking person more appealing?

Formal Definition of Dot Product

Single word that parallels "Recent" when discussing the near future

Would life always name the light from their sun "white"

Why are solar panels kept tilted?

White foam around tubeless tires

Why does lemon juice reduce the "fish" odor of sea food — specifically fish?

Show solution to recurrence is never a square

How to cope with regret and shame about not fully utilizing opportunities during PhD?

What dog breeds survive the apocalypse for generations?

Do we have C++20 ranges library in GCC 9?

Uh oh, the propeller fell off

Use of さ as a filler



Series that evaluates to different values


What is the sum of the infinite telescopic series $sum_n=1^infty arctan (n+4)-arctan(n+2))$?Convergence of the series using power seriesSimplify this summation: $sumlimits_k=5^inftybinomk-1k-5frack^32^k$Convergent series with general term $a_nb_n$Decide whether the series $sum_n=1^infty frac1+5^n1+6^n$ converges or divergesCan different choices of regulator assign different values to the same divergent series?Riemann series theorem - when can I be sure that my series diverges?Find for which values of $x$ the series convergesWorking on series $A(x):=sum_n=0^infty2^-ncos(nx)$Expected value of an r.v. that counts the number of pickable values that never appear in a random sequence













8












$begingroup$


I am trying to find a sequence $x_pq$ such that $sum_p=1^inftysum_q=1^infty x_pq$ and $sum_q=1^inftysum_p=1^infty x_pq$ both exist and evaluate to different values. I am thinking of this as a matrix, but I can't seem to see how adding all the elements of it column wise vs row-wise would change the total sum? Any hints/examples would be helpful.










share|cite|improve this question











$endgroup$







  • 1




    $begingroup$
    Is this the kind of thing you're looking for? The second paragraph gives an example and the convergent values for which a series can be rearranged and obtain different values.
    $endgroup$
    – Clayton
    May 9 at 21:52







  • 1




    $begingroup$
    This theorem is what motivates my question - but how would you write an explicit example with 2 indices?
    $endgroup$
    – user672611
    May 9 at 22:00










  • $begingroup$
    One of the classical example (which should be well-attested enough to find a detailed proof for) is the Eisenstein series $G_2(tau) = sum_m, n (n + m tau)^-2$. Unlike the higher $G_n$, swapping the order of the summation $n, m$ produces an extra $2pi i/tau$ term.
    $endgroup$
    – anomaly
    May 10 at 2:41















8












$begingroup$


I am trying to find a sequence $x_pq$ such that $sum_p=1^inftysum_q=1^infty x_pq$ and $sum_q=1^inftysum_p=1^infty x_pq$ both exist and evaluate to different values. I am thinking of this as a matrix, but I can't seem to see how adding all the elements of it column wise vs row-wise would change the total sum? Any hints/examples would be helpful.










share|cite|improve this question











$endgroup$







  • 1




    $begingroup$
    Is this the kind of thing you're looking for? The second paragraph gives an example and the convergent values for which a series can be rearranged and obtain different values.
    $endgroup$
    – Clayton
    May 9 at 21:52







  • 1




    $begingroup$
    This theorem is what motivates my question - but how would you write an explicit example with 2 indices?
    $endgroup$
    – user672611
    May 9 at 22:00










  • $begingroup$
    One of the classical example (which should be well-attested enough to find a detailed proof for) is the Eisenstein series $G_2(tau) = sum_m, n (n + m tau)^-2$. Unlike the higher $G_n$, swapping the order of the summation $n, m$ produces an extra $2pi i/tau$ term.
    $endgroup$
    – anomaly
    May 10 at 2:41













8












8








8


2



$begingroup$


I am trying to find a sequence $x_pq$ such that $sum_p=1^inftysum_q=1^infty x_pq$ and $sum_q=1^inftysum_p=1^infty x_pq$ both exist and evaluate to different values. I am thinking of this as a matrix, but I can't seem to see how adding all the elements of it column wise vs row-wise would change the total sum? Any hints/examples would be helpful.










share|cite|improve this question











$endgroup$




I am trying to find a sequence $x_pq$ such that $sum_p=1^inftysum_q=1^infty x_pq$ and $sum_q=1^inftysum_p=1^infty x_pq$ both exist and evaluate to different values. I am thinking of this as a matrix, but I can't seem to see how adding all the elements of it column wise vs row-wise would change the total sum? Any hints/examples would be helpful.







real-analysis sequences-and-series summation






share|cite|improve this question















share|cite|improve this question













share|cite|improve this question




share|cite|improve this question








edited May 10 at 19:02

























asked May 9 at 21:47







user672611














  • 1




    $begingroup$
    Is this the kind of thing you're looking for? The second paragraph gives an example and the convergent values for which a series can be rearranged and obtain different values.
    $endgroup$
    – Clayton
    May 9 at 21:52







  • 1




    $begingroup$
    This theorem is what motivates my question - but how would you write an explicit example with 2 indices?
    $endgroup$
    – user672611
    May 9 at 22:00










  • $begingroup$
    One of the classical example (which should be well-attested enough to find a detailed proof for) is the Eisenstein series $G_2(tau) = sum_m, n (n + m tau)^-2$. Unlike the higher $G_n$, swapping the order of the summation $n, m$ produces an extra $2pi i/tau$ term.
    $endgroup$
    – anomaly
    May 10 at 2:41












  • 1




    $begingroup$
    Is this the kind of thing you're looking for? The second paragraph gives an example and the convergent values for which a series can be rearranged and obtain different values.
    $endgroup$
    – Clayton
    May 9 at 21:52







  • 1




    $begingroup$
    This theorem is what motivates my question - but how would you write an explicit example with 2 indices?
    $endgroup$
    – user672611
    May 9 at 22:00










  • $begingroup$
    One of the classical example (which should be well-attested enough to find a detailed proof for) is the Eisenstein series $G_2(tau) = sum_m, n (n + m tau)^-2$. Unlike the higher $G_n$, swapping the order of the summation $n, m$ produces an extra $2pi i/tau$ term.
    $endgroup$
    – anomaly
    May 10 at 2:41







1




1




$begingroup$
Is this the kind of thing you're looking for? The second paragraph gives an example and the convergent values for which a series can be rearranged and obtain different values.
$endgroup$
– Clayton
May 9 at 21:52





$begingroup$
Is this the kind of thing you're looking for? The second paragraph gives an example and the convergent values for which a series can be rearranged and obtain different values.
$endgroup$
– Clayton
May 9 at 21:52





1




1




$begingroup$
This theorem is what motivates my question - but how would you write an explicit example with 2 indices?
$endgroup$
– user672611
May 9 at 22:00




$begingroup$
This theorem is what motivates my question - but how would you write an explicit example with 2 indices?
$endgroup$
– user672611
May 9 at 22:00












$begingroup$
One of the classical example (which should be well-attested enough to find a detailed proof for) is the Eisenstein series $G_2(tau) = sum_m, n (n + m tau)^-2$. Unlike the higher $G_n$, swapping the order of the summation $n, m$ produces an extra $2pi i/tau$ term.
$endgroup$
– anomaly
May 10 at 2:41




$begingroup$
One of the classical example (which should be well-attested enough to find a detailed proof for) is the Eisenstein series $G_2(tau) = sum_m, n (n + m tau)^-2$. Unlike the higher $G_n$, swapping the order of the summation $n, m$ produces an extra $2pi i/tau$ term.
$endgroup$
– anomaly
May 10 at 2:41










2 Answers
2






active

oldest

votes


















6












$begingroup$

Maybe you are looking for something like this: $$beginpmatrix1 &-1 & 0&0&0 & 0 &cdots \
0 & 1 & -1& 0 & 0 & 0&cdots\
0 & 0 & 1 & -1 & 0 & 0&cdots\
0 & 0 & 0 & 1 & -1 & 0&cdots \
0 & 0 & 0 & 0 & 1 & -1&cdots \
vdots &vdots &vdots & vdots & vdots & ddots&ddots
endpmatrix.$$



So that $x_p,p = 1$, $x_p,p+1=-1$ and $x_p,q = 0$ if $q neq p,p+1$.



Here we have $$sum_q=1^infty x_p,q = 0$$ for all $p in mathbb N$, and so $$sum^infty_p=1 sum^infty_q=1 x_p,q = 0.$$ However, we also have $$sum_p=1^infty x_p,q = 0 ,,,, text for ,,,, q ge 2, ,,,,, text but ,,,,, sum_p=1^infty x_p,1 = 1$$ and so $$sum^infty_q=1 sum^infty_p=1 x_p,q = 1.$$



As was said in the other answer, if $$sum_p,q lvert x_p,qrvert$$ converges (which isn't the case here), then Fubini's theorem tells us we can sum in either dimension first and get the same answer. Similarly, if all $x_p,q >0$, we can do the same, by Tonelli's theorem and this allows for the possibility that the sums evaluate to $+infty$.






share|cite|improve this answer











$endgroup$












  • $begingroup$
    Thats a good example where the sums are not just of different sign (+1).
    $endgroup$
    – RRL
    May 9 at 22:53


















5












$begingroup$

The iterated sums will be equal (if they converge) when all terms are nonnegative or, more generally, when we have absolute convergence.



Consider, on the other hand, $a_jk = 1/(k^2-j^2)$ if $jneq k$ and $a_jk = 0$ if $j=k$.



Here we have,



$$fracpi^28=tag*sum_j=1^infty sum_k=1\jneq k^infty frac1k^2 - j^2 neq sum_k=1^infty sum_j=1\j neq k^infty frac1k^2-j^2 = -fracpi^28 $$



To obtain the sum, note that



$$sum_k=1\jneq k^K frac1k^2-j^2 = frac12jsum_k=1\jneq k^K left(frac1k-j - frac1k+j right) \= frac12jleft(-sum_k=1^j-1frac1k + sum_k=1^K-jfrac1k - sum_k=j+1^K+jfrac1k + frac12j right)\= frac12jleft(frac1j +sum_k=j+1^K-jfrac1k - sum_k=j+1^K+jfrac1k + frac12j right)\ = frac12jleft(frac32j - sum_k=-j+1^j frac1K+kright) $$



and,



$$sum_k=1\jneq k^infty frac1k^2-j^2 = lim_K to inftyfrac12jleft(frac32j - sum_k=-n+1^n frac1K+kright) = frac34j^2$$



Thus,



$$sum_j=1^infty sum_k=1\jneq k^infty frac1k^2 - j^2 = sum_j=1^infty frac34j^2 = frac34 fracpi^26$$






share|cite|improve this answer









$endgroup$













    Your Answer








    StackExchange.ready(function()
    var channelOptions =
    tags: "".split(" "),
    id: "69"
    ;
    initTagRenderer("".split(" "), "".split(" "), channelOptions);

    StackExchange.using("externalEditor", function()
    // Have to fire editor after snippets, if snippets enabled
    if (StackExchange.settings.snippets.snippetsEnabled)
    StackExchange.using("snippets", function()
    createEditor();
    );

    else
    createEditor();

    );

    function createEditor()
    StackExchange.prepareEditor(
    heartbeatType: 'answer',
    autoActivateHeartbeat: false,
    convertImagesToLinks: true,
    noModals: true,
    showLowRepImageUploadWarning: true,
    reputationToPostImages: 10,
    bindNavPrevention: true,
    postfix: "",
    imageUploader:
    brandingHtml: "Powered by u003ca class="icon-imgur-white" href="https://imgur.com/"u003eu003c/au003e",
    contentPolicyHtml: "User contributions licensed under u003ca href="https://creativecommons.org/licenses/by-sa/3.0/"u003ecc by-sa 3.0 with attribution requiredu003c/au003e u003ca href="https://stackoverflow.com/legal/content-policy"u003e(content policy)u003c/au003e",
    allowUrls: true
    ,
    noCode: true, onDemand: true,
    discardSelector: ".discard-answer"
    ,immediatelyShowMarkdownHelp:true
    );



    );













    draft saved

    draft discarded


















    StackExchange.ready(
    function ()
    StackExchange.openid.initPostLogin('.new-post-login', 'https%3a%2f%2fmath.stackexchange.com%2fquestions%2f3220297%2fseries-that-evaluates-to-different-values%23new-answer', 'question_page');

    );

    Post as a guest















    Required, but never shown
























    2 Answers
    2






    active

    oldest

    votes








    2 Answers
    2






    active

    oldest

    votes









    active

    oldest

    votes






    active

    oldest

    votes









    6












    $begingroup$

    Maybe you are looking for something like this: $$beginpmatrix1 &-1 & 0&0&0 & 0 &cdots \
    0 & 1 & -1& 0 & 0 & 0&cdots\
    0 & 0 & 1 & -1 & 0 & 0&cdots\
    0 & 0 & 0 & 1 & -1 & 0&cdots \
    0 & 0 & 0 & 0 & 1 & -1&cdots \
    vdots &vdots &vdots & vdots & vdots & ddots&ddots
    endpmatrix.$$



    So that $x_p,p = 1$, $x_p,p+1=-1$ and $x_p,q = 0$ if $q neq p,p+1$.



    Here we have $$sum_q=1^infty x_p,q = 0$$ for all $p in mathbb N$, and so $$sum^infty_p=1 sum^infty_q=1 x_p,q = 0.$$ However, we also have $$sum_p=1^infty x_p,q = 0 ,,,, text for ,,,, q ge 2, ,,,,, text but ,,,,, sum_p=1^infty x_p,1 = 1$$ and so $$sum^infty_q=1 sum^infty_p=1 x_p,q = 1.$$



    As was said in the other answer, if $$sum_p,q lvert x_p,qrvert$$ converges (which isn't the case here), then Fubini's theorem tells us we can sum in either dimension first and get the same answer. Similarly, if all $x_p,q >0$, we can do the same, by Tonelli's theorem and this allows for the possibility that the sums evaluate to $+infty$.






    share|cite|improve this answer











    $endgroup$












    • $begingroup$
      Thats a good example where the sums are not just of different sign (+1).
      $endgroup$
      – RRL
      May 9 at 22:53















    6












    $begingroup$

    Maybe you are looking for something like this: $$beginpmatrix1 &-1 & 0&0&0 & 0 &cdots \
    0 & 1 & -1& 0 & 0 & 0&cdots\
    0 & 0 & 1 & -1 & 0 & 0&cdots\
    0 & 0 & 0 & 1 & -1 & 0&cdots \
    0 & 0 & 0 & 0 & 1 & -1&cdots \
    vdots &vdots &vdots & vdots & vdots & ddots&ddots
    endpmatrix.$$



    So that $x_p,p = 1$, $x_p,p+1=-1$ and $x_p,q = 0$ if $q neq p,p+1$.



    Here we have $$sum_q=1^infty x_p,q = 0$$ for all $p in mathbb N$, and so $$sum^infty_p=1 sum^infty_q=1 x_p,q = 0.$$ However, we also have $$sum_p=1^infty x_p,q = 0 ,,,, text for ,,,, q ge 2, ,,,,, text but ,,,,, sum_p=1^infty x_p,1 = 1$$ and so $$sum^infty_q=1 sum^infty_p=1 x_p,q = 1.$$



    As was said in the other answer, if $$sum_p,q lvert x_p,qrvert$$ converges (which isn't the case here), then Fubini's theorem tells us we can sum in either dimension first and get the same answer. Similarly, if all $x_p,q >0$, we can do the same, by Tonelli's theorem and this allows for the possibility that the sums evaluate to $+infty$.






    share|cite|improve this answer











    $endgroup$












    • $begingroup$
      Thats a good example where the sums are not just of different sign (+1).
      $endgroup$
      – RRL
      May 9 at 22:53













    6












    6








    6





    $begingroup$

    Maybe you are looking for something like this: $$beginpmatrix1 &-1 & 0&0&0 & 0 &cdots \
    0 & 1 & -1& 0 & 0 & 0&cdots\
    0 & 0 & 1 & -1 & 0 & 0&cdots\
    0 & 0 & 0 & 1 & -1 & 0&cdots \
    0 & 0 & 0 & 0 & 1 & -1&cdots \
    vdots &vdots &vdots & vdots & vdots & ddots&ddots
    endpmatrix.$$



    So that $x_p,p = 1$, $x_p,p+1=-1$ and $x_p,q = 0$ if $q neq p,p+1$.



    Here we have $$sum_q=1^infty x_p,q = 0$$ for all $p in mathbb N$, and so $$sum^infty_p=1 sum^infty_q=1 x_p,q = 0.$$ However, we also have $$sum_p=1^infty x_p,q = 0 ,,,, text for ,,,, q ge 2, ,,,,, text but ,,,,, sum_p=1^infty x_p,1 = 1$$ and so $$sum^infty_q=1 sum^infty_p=1 x_p,q = 1.$$



    As was said in the other answer, if $$sum_p,q lvert x_p,qrvert$$ converges (which isn't the case here), then Fubini's theorem tells us we can sum in either dimension first and get the same answer. Similarly, if all $x_p,q >0$, we can do the same, by Tonelli's theorem and this allows for the possibility that the sums evaluate to $+infty$.






    share|cite|improve this answer











    $endgroup$



    Maybe you are looking for something like this: $$beginpmatrix1 &-1 & 0&0&0 & 0 &cdots \
    0 & 1 & -1& 0 & 0 & 0&cdots\
    0 & 0 & 1 & -1 & 0 & 0&cdots\
    0 & 0 & 0 & 1 & -1 & 0&cdots \
    0 & 0 & 0 & 0 & 1 & -1&cdots \
    vdots &vdots &vdots & vdots & vdots & ddots&ddots
    endpmatrix.$$



    So that $x_p,p = 1$, $x_p,p+1=-1$ and $x_p,q = 0$ if $q neq p,p+1$.



    Here we have $$sum_q=1^infty x_p,q = 0$$ for all $p in mathbb N$, and so $$sum^infty_p=1 sum^infty_q=1 x_p,q = 0.$$ However, we also have $$sum_p=1^infty x_p,q = 0 ,,,, text for ,,,, q ge 2, ,,,,, text but ,,,,, sum_p=1^infty x_p,1 = 1$$ and so $$sum^infty_q=1 sum^infty_p=1 x_p,q = 1.$$



    As was said in the other answer, if $$sum_p,q lvert x_p,qrvert$$ converges (which isn't the case here), then Fubini's theorem tells us we can sum in either dimension first and get the same answer. Similarly, if all $x_p,q >0$, we can do the same, by Tonelli's theorem and this allows for the possibility that the sums evaluate to $+infty$.







    share|cite|improve this answer














    share|cite|improve this answer



    share|cite|improve this answer








    edited May 10 at 0:50









    Brian Borchers

    6,55011320




    6,55011320










    answered May 9 at 22:51









    User8128User8128

    11.2k1622




    11.2k1622











    • $begingroup$
      Thats a good example where the sums are not just of different sign (+1).
      $endgroup$
      – RRL
      May 9 at 22:53
















    • $begingroup$
      Thats a good example where the sums are not just of different sign (+1).
      $endgroup$
      – RRL
      May 9 at 22:53















    $begingroup$
    Thats a good example where the sums are not just of different sign (+1).
    $endgroup$
    – RRL
    May 9 at 22:53




    $begingroup$
    Thats a good example where the sums are not just of different sign (+1).
    $endgroup$
    – RRL
    May 9 at 22:53











    5












    $begingroup$

    The iterated sums will be equal (if they converge) when all terms are nonnegative or, more generally, when we have absolute convergence.



    Consider, on the other hand, $a_jk = 1/(k^2-j^2)$ if $jneq k$ and $a_jk = 0$ if $j=k$.



    Here we have,



    $$fracpi^28=tag*sum_j=1^infty sum_k=1\jneq k^infty frac1k^2 - j^2 neq sum_k=1^infty sum_j=1\j neq k^infty frac1k^2-j^2 = -fracpi^28 $$



    To obtain the sum, note that



    $$sum_k=1\jneq k^K frac1k^2-j^2 = frac12jsum_k=1\jneq k^K left(frac1k-j - frac1k+j right) \= frac12jleft(-sum_k=1^j-1frac1k + sum_k=1^K-jfrac1k - sum_k=j+1^K+jfrac1k + frac12j right)\= frac12jleft(frac1j +sum_k=j+1^K-jfrac1k - sum_k=j+1^K+jfrac1k + frac12j right)\ = frac12jleft(frac32j - sum_k=-j+1^j frac1K+kright) $$



    and,



    $$sum_k=1\jneq k^infty frac1k^2-j^2 = lim_K to inftyfrac12jleft(frac32j - sum_k=-n+1^n frac1K+kright) = frac34j^2$$



    Thus,



    $$sum_j=1^infty sum_k=1\jneq k^infty frac1k^2 - j^2 = sum_j=1^infty frac34j^2 = frac34 fracpi^26$$






    share|cite|improve this answer









    $endgroup$

















      5












      $begingroup$

      The iterated sums will be equal (if they converge) when all terms are nonnegative or, more generally, when we have absolute convergence.



      Consider, on the other hand, $a_jk = 1/(k^2-j^2)$ if $jneq k$ and $a_jk = 0$ if $j=k$.



      Here we have,



      $$fracpi^28=tag*sum_j=1^infty sum_k=1\jneq k^infty frac1k^2 - j^2 neq sum_k=1^infty sum_j=1\j neq k^infty frac1k^2-j^2 = -fracpi^28 $$



      To obtain the sum, note that



      $$sum_k=1\jneq k^K frac1k^2-j^2 = frac12jsum_k=1\jneq k^K left(frac1k-j - frac1k+j right) \= frac12jleft(-sum_k=1^j-1frac1k + sum_k=1^K-jfrac1k - sum_k=j+1^K+jfrac1k + frac12j right)\= frac12jleft(frac1j +sum_k=j+1^K-jfrac1k - sum_k=j+1^K+jfrac1k + frac12j right)\ = frac12jleft(frac32j - sum_k=-j+1^j frac1K+kright) $$



      and,



      $$sum_k=1\jneq k^infty frac1k^2-j^2 = lim_K to inftyfrac12jleft(frac32j - sum_k=-n+1^n frac1K+kright) = frac34j^2$$



      Thus,



      $$sum_j=1^infty sum_k=1\jneq k^infty frac1k^2 - j^2 = sum_j=1^infty frac34j^2 = frac34 fracpi^26$$






      share|cite|improve this answer









      $endgroup$















        5












        5








        5





        $begingroup$

        The iterated sums will be equal (if they converge) when all terms are nonnegative or, more generally, when we have absolute convergence.



        Consider, on the other hand, $a_jk = 1/(k^2-j^2)$ if $jneq k$ and $a_jk = 0$ if $j=k$.



        Here we have,



        $$fracpi^28=tag*sum_j=1^infty sum_k=1\jneq k^infty frac1k^2 - j^2 neq sum_k=1^infty sum_j=1\j neq k^infty frac1k^2-j^2 = -fracpi^28 $$



        To obtain the sum, note that



        $$sum_k=1\jneq k^K frac1k^2-j^2 = frac12jsum_k=1\jneq k^K left(frac1k-j - frac1k+j right) \= frac12jleft(-sum_k=1^j-1frac1k + sum_k=1^K-jfrac1k - sum_k=j+1^K+jfrac1k + frac12j right)\= frac12jleft(frac1j +sum_k=j+1^K-jfrac1k - sum_k=j+1^K+jfrac1k + frac12j right)\ = frac12jleft(frac32j - sum_k=-j+1^j frac1K+kright) $$



        and,



        $$sum_k=1\jneq k^infty frac1k^2-j^2 = lim_K to inftyfrac12jleft(frac32j - sum_k=-n+1^n frac1K+kright) = frac34j^2$$



        Thus,



        $$sum_j=1^infty sum_k=1\jneq k^infty frac1k^2 - j^2 = sum_j=1^infty frac34j^2 = frac34 fracpi^26$$






        share|cite|improve this answer









        $endgroup$



        The iterated sums will be equal (if they converge) when all terms are nonnegative or, more generally, when we have absolute convergence.



        Consider, on the other hand, $a_jk = 1/(k^2-j^2)$ if $jneq k$ and $a_jk = 0$ if $j=k$.



        Here we have,



        $$fracpi^28=tag*sum_j=1^infty sum_k=1\jneq k^infty frac1k^2 - j^2 neq sum_k=1^infty sum_j=1\j neq k^infty frac1k^2-j^2 = -fracpi^28 $$



        To obtain the sum, note that



        $$sum_k=1\jneq k^K frac1k^2-j^2 = frac12jsum_k=1\jneq k^K left(frac1k-j - frac1k+j right) \= frac12jleft(-sum_k=1^j-1frac1k + sum_k=1^K-jfrac1k - sum_k=j+1^K+jfrac1k + frac12j right)\= frac12jleft(frac1j +sum_k=j+1^K-jfrac1k - sum_k=j+1^K+jfrac1k + frac12j right)\ = frac12jleft(frac32j - sum_k=-j+1^j frac1K+kright) $$



        and,



        $$sum_k=1\jneq k^infty frac1k^2-j^2 = lim_K to inftyfrac12jleft(frac32j - sum_k=-n+1^n frac1K+kright) = frac34j^2$$



        Thus,



        $$sum_j=1^infty sum_k=1\jneq k^infty frac1k^2 - j^2 = sum_j=1^infty frac34j^2 = frac34 fracpi^26$$







        share|cite|improve this answer












        share|cite|improve this answer



        share|cite|improve this answer










        answered May 9 at 22:47









        RRLRRL

        55k52776




        55k52776



























            draft saved

            draft discarded
















































            Thanks for contributing an answer to Mathematics Stack Exchange!


            • Please be sure to answer the question. Provide details and share your research!

            But avoid


            • Asking for help, clarification, or responding to other answers.

            • Making statements based on opinion; back them up with references or personal experience.

            Use MathJax to format equations. MathJax reference.


            To learn more, see our tips on writing great answers.




            draft saved


            draft discarded














            StackExchange.ready(
            function ()
            StackExchange.openid.initPostLogin('.new-post-login', 'https%3a%2f%2fmath.stackexchange.com%2fquestions%2f3220297%2fseries-that-evaluates-to-different-values%23new-answer', 'question_page');

            );

            Post as a guest















            Required, but never shown





















































            Required, but never shown














            Required, but never shown












            Required, but never shown







            Required, but never shown

































            Required, but never shown














            Required, but never shown












            Required, but never shown







            Required, but never shown







            Popular posts from this blog

            Get product attribute by attribute group code in magento 2get product attribute by product attribute group in magento 2Magento 2 Log Bundle Product Data in List Page?How to get all product attribute of a attribute group of Default attribute set?Magento 2.1 Create a filter in the product grid by new attributeMagento 2 : Get Product Attribute values By GroupMagento 2 How to get all existing values for one attributeMagento 2 get custom attribute of a single product inside a pluginMagento 2.3 How to get all the Multi Source Inventory (MSI) locations collection in custom module?Magento2: how to develop rest API to get new productsGet product attribute by attribute group code ( [attribute_group_code] ) in magento 2

            Category:9 (number) SubcategoriesMedia in category "9 (number)"Navigation menuUpload mediaGND ID: 4485639-8Library of Congress authority ID: sh85091979ReasonatorScholiaStatistics

            Magento 2.3: How do i solve this, Not registered handle, on custom form?How can i rewrite TierPrice Block in Magento2magento 2 captcha not rendering if I override layout xmlmain.CRITICAL: Plugin class doesn't existMagento 2 : Problem while adding custom button order view page?Magento 2.2.5: Overriding Admin Controller sales/orderMagento 2.2.5: Add, Update and Delete existing products Custom OptionsMagento 2.3 : File Upload issue in UI Component FormMagento2 Not registered handleHow to configured Form Builder Js in my custom magento 2.3.0 module?Magento 2.3. How to create image upload field in an admin form